What I forgot to mention is that I need the filter to make it work.

When you mentioned that filter, it raised suspicion to me. What exactly is that filter? I wasn't able to find any information on it.

Does the system work OK with the filter removed from the equation?

___________
Tony Fabris
_________________________
Tony Fabris